alfa problems

Algebra Level 3

If a 1 , a 2 , a 3 . . . . . . . a 2015 > 0 , a 1 + a 2 + . . . + a 2015 = 1 a_{1},a_{2},a_{3}.......a_{2015} > 0, a_{1} + a_{2} + ... + a_{2015} = 1

and the minimum value of

1 a 1 + 1 a 2 + 1 a 3 + . . . + 1 a 2015 = x \frac{1}{{a}_{1}} + \frac{1}{{a}_{2}} + \frac{1}{{a}_{3}} + ... + \frac{1}{{a}_{2015}} = x

what is the value of x \sqrt{x}


The answer is 2015.

This section requires Javascript.
You are seeing this because something didn't load right. We suggest you, (a) try refreshing the page, (b) enabling javascript if it is disabled on your browser and, finally, (c) loading the non-javascript version of this page . We're sorry about the hassle.

3 solutions

Satyendra Kumar
Dec 7, 2014

L e t u s u s e A . M H . M : Let\ us\ use\ A.M-H.M: 1 a 1 + 1 a 2 + 1 a 3 + . . . 1 a 2015 2015 2015 a 1 + a 2 + a 3 + . . . . a 2015 \dfrac{\dfrac{1}{a_1}+\dfrac{1}{a_2}+\dfrac{1}{a_3}+...\dfrac{1}{a_{2015}}}{2015}\geq\dfrac{2015}{a_1+a_2+a_3+....a_{2015}} B u t , a 1 + a 2 + a 3 + . . . . a 2015 = 1 But,a_1+a_2+a_3+....a_{2015}=1 1 a 1 + 1 a 2 + 1 a 3 + . . . 1 a 2015 2015 2015 \Longrightarrow\ \dfrac{\dfrac{1}{a_1}+\dfrac{1}{a_2}+\dfrac{1}{a_3}+...\dfrac{1}{a_{2015}}}{2015}\geq2015 1 a 1 + 1 a 2 + 1 a 3 + . . . 1 a 2015 201 5 2 \Longrightarrow\ \dfrac{1}{a_1}+\dfrac{1}{a_2}+\dfrac{1}{a_3}+...\dfrac{1}{a_{2015}}\geq2015^{2} a n s w e r = 201 5 2 = 2015. \Longrightarrow\ answer=\sqrt{2015^{2}}=2015. A . M H . M w o r k s t h i s w a y : ( a 1 + a 2 + . . . a n ) n n 1 a 1 + 1 a 2 + . . . 1 a n A.M-H.M\ works\ this\ way:\dfrac{(a_1+a_2+...a_n)}{n}\geq\dfrac{n}{\dfrac{1}{a_1}+\dfrac{1}{a_2}+...\dfrac{1}{a_n}}

Deepanshu Gupta
Dec 8, 2014

we can used Jenson's inequality

Let y = f ( x ) = 1 x x ( 0 , 1 ) y\quad =\quad f\left( x \right) \quad =\quad \cfrac { 1 }{ x } \quad \quad \quad \quad \quad x\quad \in \quad (0,1) .

Pic Pic

Consider 2015 point's on the curve So after joining them an polygon is formed whose Centroid is G and which lies inside the polygon , and a point P which lies on the curve with same x-coordinate as that of centroid So

y G y p f ( x i ) n f ( x i n ) f ( x i ) n f ( 1 n ) ( p u t n = 2015 ) f ( x i ) 2015 2 k = 2015 { y }_{ G }\quad \ge \quad { y }_{ p }\\ \\ \frac { \sum { f\left( { x }_{ i } \right) } }{ n } \quad \ge \quad f\left( \cfrac { \sum { { x }_{ i } } }{ n } \right) \\ \\ \sum { f\left( { x }_{ i } \right) } \quad \ge \quad nf\left( \cfrac { 1 }{ n } \right) \quad \quad \quad (put\quad n\quad =\quad 2015)\\ \\ \sum { f\left( { x }_{ i } \right) } \quad \ge \quad { 2015 }^{ 2 }\\ \\ \boxed { k\quad =\quad 2015 } .

How did you include the graph in your solution? :o

Aritra Jana - 6 years, 6 months ago

Log in to reply

hi Aritra , First Upload your image on the sites Like : imgur,com

and then Copy the image URL ( By right click on image )

and Use following code at brilliant :

! [Type Title] (Post image URL here)

Don't give Space between two brackets and exclamation ( ! ) mark

Deepanshu Gupta - 6 years, 6 months ago

Log in to reply

thanks a ton !! :D

Aritra Jana - 6 years, 6 months ago

It can also be done by Titu's Lemma

Shivam Hinduja - 6 years, 6 months ago
Ryan Tamburrino
Dec 12, 2014

I did this through a simple application of Titu's Lemma. The key is to treat all the numerators as if they were the square of 1, rather than just 1. Easiest solution, in my opinion.

@Ryan Tamburrino can you please write easiest solution in your mind style.

Zeeshan Memon - 6 years, 4 months ago

0 pending reports

×

Problem Loading...

Note Loading...

Set Loading...